- Mon Jan 20, 2014 12:00 am
#41621
Complete Question Explanation
(The complete setup for this game can be found here: lsat/viewtopic.php?t=15887)
The correct answer choice is (E)
This Justify question asks us to identify a condition that, if true, would fully determine the order in which the students perform. Questions of this type can be particularly challenging if you have not previously examined the variables (and rules) that are especially likely to restrict the solution of the game.
Let’s examine our initial diagram for points of restriction. Notice, for instance, the two dual options we were able to infer: either G or K must be first, and either H or F must be fourth. To fully determine the order in which the students perform, the correct answer choice must “settle” these dual options. In other words, it must determine—at the very least—whether K or G performs first, and also whether H or F performs fourth.
Answer choice (A): This answer choice is incorrect. Knowing the precise order of the variables in the HF rotating block is not enough to fully determine the order of all five variables, if only because we still do not know if K or G performs first, nor can we determine the placement of J.
Answer choice (B): This answer choice is attractive, because a GF block necessitates the inclusion of H immediately after F, thanks to the third rule. And, since G cannot perform first in this instance, K must be first. This solution seems promising, especially if you consider the local diagram for Question #2. In that diagram, a GFH block can fully determine the order of the five performances:
Recall, however, that the diagram for Question #2 was predicated on the condition that J performs earlier than G. However, what if J performs later than G? A GFH block would still be possible, but the solution would be different:
Clearly, then, a GF block is not sufficient to fully determine the order of all five of the performances, rendering answer choice (B) incorrect.
Answer choice (C): This answer choice is also attractive, because the last rule requires the inclusion of F immediately before H in the HJ block, creating a powerful FHJ block. However, the relative order of K and G is still unknown, which renders answer choice (C) incorrect.
Answer choice (D): This answer choice is similar to answer choice (C). The last rule requires the inclusion of F immediately after H in the JH block, creating a JHF block. Nevertheless, the relative order of K and G is still unknown, which renders answer choice (D) also incorrect.
Answer choice (E): This is the correct answer choice. If K performs immediately before F, the last rule creates a KFH block. Thanks to the first rule, G must perform before the KFH block. Additionally, since K must perform earlier than J (second rule), it follows that J must perform last:
Since answer choice (E) is sufficient to fully determine the order in which the students perform, it is the correct answer choice.
Access to the two templates outlined earlier would have made our decision even easier. Since only Template 1 allows for a KF block, the solution is already restricted to one of the two templates:
It should be clear from this Template that a KF block is only possible if K is second, and F—third. This resolves all dual options in that setup, proving that the game has only one possible solution.
(The complete setup for this game can be found here: lsat/viewtopic.php?t=15887)
The correct answer choice is (E)
This Justify question asks us to identify a condition that, if true, would fully determine the order in which the students perform. Questions of this type can be particularly challenging if you have not previously examined the variables (and rules) that are especially likely to restrict the solution of the game.
Let’s examine our initial diagram for points of restriction. Notice, for instance, the two dual options we were able to infer: either G or K must be first, and either H or F must be fourth. To fully determine the order in which the students perform, the correct answer choice must “settle” these dual options. In other words, it must determine—at the very least—whether K or G performs first, and also whether H or F performs fourth.
Answer choice (A): This answer choice is incorrect. Knowing the precise order of the variables in the HF rotating block is not enough to fully determine the order of all five variables, if only because we still do not know if K or G performs first, nor can we determine the placement of J.
Answer choice (B): This answer choice is attractive, because a GF block necessitates the inclusion of H immediately after F, thanks to the third rule. And, since G cannot perform first in this instance, K must be first. This solution seems promising, especially if you consider the local diagram for Question #2. In that diagram, a GFH block can fully determine the order of the five performances:
Recall, however, that the diagram for Question #2 was predicated on the condition that J performs earlier than G. However, what if J performs later than G? A GFH block would still be possible, but the solution would be different:
Clearly, then, a GF block is not sufficient to fully determine the order of all five of the performances, rendering answer choice (B) incorrect.
Answer choice (C): This answer choice is also attractive, because the last rule requires the inclusion of F immediately before H in the HJ block, creating a powerful FHJ block. However, the relative order of K and G is still unknown, which renders answer choice (C) incorrect.
Answer choice (D): This answer choice is similar to answer choice (C). The last rule requires the inclusion of F immediately after H in the JH block, creating a JHF block. Nevertheless, the relative order of K and G is still unknown, which renders answer choice (D) also incorrect.
Answer choice (E): This is the correct answer choice. If K performs immediately before F, the last rule creates a KFH block. Thanks to the first rule, G must perform before the KFH block. Additionally, since K must perform earlier than J (second rule), it follows that J must perform last:
Since answer choice (E) is sufficient to fully determine the order in which the students perform, it is the correct answer choice.
Access to the two templates outlined earlier would have made our decision even easier. Since only Template 1 allows for a KF block, the solution is already restricted to one of the two templates:
It should be clear from this Template that a KF block is only possible if K is second, and F—third. This resolves all dual options in that setup, proving that the game has only one possible solution.
You do not have the required permissions to view the files attached to this post.